Proof: Convergent Sequence is Bounded | Real Analysis

Поділитися
Вставка
  • Опубліковано 15 вер 2024
  • Any convergent sequence must be bounded. We'll prove this basic result about convergent sequences in today's lesson. We use the definition of the limit of a sequence, a useful equivalence involving absolute value inequalities, and then considering a maximum and minimum will help us find an upper and lower bound to use for our proof that a convergent sequence is bounded!
    What are Bounded Sequences? • What are Bounded Seque...
    A Useful Absolute Value Inequality: • Proof: A Useful Absolu...
    Definition of the Limit of a Sequence: • Definition of the Limi...
    Real Analysis Playlist: • Real Analysis
    ★DONATE★
    ◆ Support Wrath of Math on Patreon for early access to new videos and other exclusive benefits: / wrathofmathlessons
    ◆ Donate on PayPal: www.paypal.me/...
    Thanks to Robert Rennie and Barbara Sharrock for their generous support on Patreon!
    Thanks to Crayon Angel, my favorite musician in the world, who upon my request gave me permission to use his music in my math lessons: crayonangel.ba...
    Follow Wrath of Math on...
    ● Instagram: / wrathofmathedu
    ● Facebook: / wrathofmath
    ● Twitter: / wrathofmathedu
    My Music Channel: / @emery3050

КОМЕНТАРІ • 80

  • @kunalkumar-ob4lg
    @kunalkumar-ob4lg 2 роки тому +24

    The explanation cannot be more simple and easy to understand. Thank you.

    • @WrathofMath
      @WrathofMath  2 роки тому +8

      So glad it helped, thanks for watching and check out my real analysis playlist if you're looking for more!
      ua-cam.com/play/PLztBpqftvzxWo4HxUYV58ENhxHV32Wxli.html

    • @kunalkumar-ob4lg
      @kunalkumar-ob4lg 2 роки тому +5

      @@WrathofMath The playlist is fire. It summarizes my whole course. I'm definitely gonna watch that.

  • @Nate3145-zt8rh
    @Nate3145-zt8rh 2 місяці тому +1

    I dont care how many times you have done this proof, if you have even a slight difficulty recalling it, do it again. It is so fundamental to analysis.

    • @Nate3145-zt8rh
      @Nate3145-zt8rh 2 місяці тому +1

      what i mean is that this proof should be intuitive and natural

  • @siriuss_
    @siriuss_ Рік тому +2

    Thanks for the video! Also, I'm trying to show that a cauchy sequence then it is bounded (similar to the video) we have, if epsilon= 1, then |a(n) - a(k) < 1. This is similar to -1 < a(n) - a(k) < 1 => a(k) -1 < a(n) + 1. Also, -(a(k) + 1) < a(k) - 1. So we have, -(a(k) + 1) < an < a(k) + 1.If, n = {1,..,N} then a(k) ≥ a(N), so we have (Taking, a(k) + 1 = K), -K ≤ a(1),..a(N) ≤ K. Now, taking M = K + 1, we have -M < a(1),..a(N) < M. Then my question is that i know that K = max{a(1),...,a(N), 1 + a(k)). So what is M then? I mean in max{} or something like that?

    • @siriuss_
      @siriuss_ Рік тому

      This was a proof in Charles Pugh' Real analysis. I tried to fill in my details myself but now I'm confused.

    • @siriuss_
      @siriuss_ Рік тому

      Another proof that I saw in abbot's Analysis. We know that, |a(n) - a(k)| < 1, but also, a(n) ≥ a(N), so if we take k = {1,..,N}. So, |a(n)| ≥ |a(N)| then, |a(n)| > |a(N)| + 1. So, If |a(n)| > M , then we take,
      M = max{1,...,|a(N - 1)|, |a(N)| + 1}. Not |a(N)| since, |a(n)| ≥ |a(N)|. Thus we have shown that the sequence is bounded. Can you improve on my reasoning here?

  • @danfirth3017
    @danfirth3017 2 роки тому +2

    these videos have carried me through this first term of analysis

  • @abdellahomaradolf3566
    @abdellahomaradolf3566 Рік тому +2

    Your carefully thought through explanation is superb. Whenever I learn a concept from your explanations, no matter how abstract it is, I concretely understand the concept.

    • @WrathofMath
      @WrathofMath  Рік тому +1

      That's awesome to hear, thank you for watching!

  • @agrani7650
    @agrani7650 3 роки тому +3

    let's just say that you are my professor for the first semester ... LOVE YOU and YOUR HARDWORK ! Thank you

    • @WrathofMath
      @WrathofMath  2 роки тому +2

      Glad to help! Thanks for watching! If you're looking for more analysis, check out my playlist! ua-cam.com/play/PLztBpqftvzxWo4HxUYV58ENhxHV32Wxli.html

  • @Bedoroski
    @Bedoroski 9 місяців тому +1

    Thanks for so much hard work into explaining it. The textbook by Jim Cummings somehow made it a bit confusing with the inequalities

    • @WrathofMath
      @WrathofMath  9 місяців тому

      Glad to help, thanks for watching!

  • @joshbolton2782
    @joshbolton2782 Рік тому

    Dude you are so good at explaining the proof so it makes sense. Thank you man.

  • @cloudhuang6691
    @cloudhuang6691 Рік тому +1

    Sir, your lesson is very comprehensive, thank you so much.

    • @WrathofMath
      @WrathofMath  Рік тому

      Glad to help - thanks for watching!

    • @cloudhuang6691
      @cloudhuang6691 Рік тому +1

      @@WrathofMath Excuse sir, I've reconsidered this basic question again recently, and something occured to me: Is'nt there any convergent sequence blows up divergently in some of the first few "N" terms so that makes the sequence unbounded?

  • @maxpercer7119
    @maxpercer7119 2 роки тому +3

    For those using the alternate definition of bounded sequence: "a sequence (a_n) is bounded if |a_n |

  • @denglannieee6702
    @denglannieee6702 Рік тому

    OMG, this way is so much simpler than my lecture. U r sooo smart! Thank you so much.

    • @WrathofMath
      @WrathofMath  Рік тому

      I'm so glad to help, thanks for watching!

  • @TheThBe
    @TheThBe 5 місяців тому

    Doesn't it suffice to prove that |An| =< U, since it will also be bounded by -U? Excellent video nonetheless, as always!

  • @dzgnt3053
    @dzgnt3053 2 роки тому

    Thank you for the great work , i will be pleased to hear the optimal way to achieve your level in maths .

  • @federjcoo
    @federjcoo 8 місяців тому

    doesnt this work only for epsilon=1? like what if i choose epsilon=2?

  • @SoumaHsu
    @SoumaHsu Рік тому

    Very clear explanation! Helps me a lot

  • @gmingwll7985
    @gmingwll7985 2 роки тому +1

    Thanks for this deep concept sir.

  • @jingyiwang5113
    @jingyiwang5113 2 роки тому +1

    Your explanation is really clear. It is so helpful for me~Thank you so much!

    • @WrathofMath
      @WrathofMath  2 роки тому

      So glad to help, thanks for watching! Let me know if you have any questions!

  • @jonhroldandl.buenaventura7596
    @jonhroldandl.buenaventura7596 11 місяців тому +1

    Can I use ε itself instead of assigning specific value? since it is defined to be greater than 0. Thank you!

  • @chk374
    @chk374 2 роки тому +1

    Good video, thank you. Question: When arriving at the end result a_k

    • @WrathofMath
      @WrathofMath  2 роки тому +1

      Thanks for watching and great question! We know a_1,a_2,...,a_N are bounded because N is a fixed natural number. In other words, the list is finite. Remember that N comes from a_n being convergent, it is the N such that all terms of the sequence after the Nth term are within 1 of epsilon, guaranteed to exist by convergence. Does that help?

    • @chk374
      @chk374 2 роки тому

      @@WrathofMath The question is more aimed at showing that a_1,a_2,…,a_N are bounded (i understand a_1,a_2,…,a_N is a set with a finite number of elements but that doesn’t guarantee said elements are bounded), ie how do you know that a_j

    • @WrathofMath
      @WrathofMath  2 роки тому

      I'm not sure I understand. The set being finite does guarantee it's bounded, since a finite set has a max and a min, which are bounds. We are talking about sequences of real numbers, so certainly every a_j is less than infinity not only for j = 1,2,...N, but for all j. Regarding 1/|k-2| I'm also not sure what you're pointing out. When you say not all a_k are bounded do you mean each a_k is not necessarily bounded? That wouldn't make sense since each a_k is just a number. So do you mean the set of all a_k for positive integers k not equal to 2?
      Maybe you're saying "How do we know none of our a_n are something like 1/0?" The answer would be that a_n was taken to be a sequence of real numbers.

    • @chk374
      @chk374 2 роки тому

      @@WrathofMath What if my sequence is 1/(k-2)? a_2 is unbounded. Doesn’t boundedness mean a_k < oo for all k? What am i missing here

    • @chk374
      @chk374 2 роки тому +1

      is the answer here that k in Narural# implies that a_2 is not defined (and not oo) and therefore a_k is really only defined for k>2?

  • @Kira-cw1yp
    @Kira-cw1yp Місяць тому

    That was such an amazing explanation....
    Thanks buddy

    • @WrathofMath
      @WrathofMath  Місяць тому

      Glad to help, thanks for watching!

    • @Kira-cw1yp
      @Kira-cw1yp Місяць тому

      @@WrathofMathI am first year student at IIT for undergrad in computer science

    • @WrathofMath
      @WrathofMath  Місяць тому

      That's awesome - good luck, computer science is a fascinating pursuit!

  • @turokg1578
    @turokg1578 Рік тому

    this channel is a lifesaver...

    • @WrathofMath
      @WrathofMath  Рік тому +1

      So glad to help! I'm continuing to work on the analysis playlist, let me know if you have any questions!

    • @turokg1578
      @turokg1578 Рік тому +2

      @@WrathofMath is it an acceptable proof if i take M = max {|a_1|, |a_2|, ... |a_N|, |1+a|, |a-1|} and say that |a_n|

  • @anveshshrivastava2115
    @anveshshrivastava2115 9 місяців тому

    Literally, cant get a better expalination

  • @BubimedesOfficial
    @BubimedesOfficial 9 місяців тому

    Do you have to select an epsilon? Because the way I understand it, it wouldn't make a difference (since you're keeping everything else abstract)

  • @scp_in_iitb
    @scp_in_iitb Місяць тому

    Very nicely explained 👍

  • @mahmoudalbahar1641
    @mahmoudalbahar1641 3 роки тому +3

    Thanks.

  • @이주홍-d5u
    @이주홍-d5u 5 місяців тому

    good and clear proof!

  • @ezenaboifeyinwa
    @ezenaboifeyinwa Рік тому

    When u were explaining for n = 1,2,3,...,n u said that it's possible for those n to be bigger than 1+a please I don't understand how 🙏

  • @xssavage7104
    @xssavage7104 Рік тому

    I'm only finding out about this channel 2 hours before an analysis exam... it's already late

  • @mathandsciencetips
    @mathandsciencetips 3 роки тому +1

    Good explaination

    • @WrathofMath
      @WrathofMath  3 роки тому +1

      Thanks, glad it was clear! Let me know if you ever have any video requests, and if you're looking for more analysis - check out my playlist: ua-cam.com/play/PLztBpqftvzxWo4HxUYV58ENhxHV32Wxli.html

  • @sanasunu7
    @sanasunu7 2 роки тому

    Tnx for ur classs.. its helpful for ma xam

    • @WrathofMath
      @WrathofMath  2 роки тому +1

      So glad to help! Thanks for watching and check out my analysis playlist if you're looking for more: ua-cam.com/play/PLztBpqftvzxWo4HxUYV58ENhxHV32Wxli.html

    • @sanasunu7
      @sanasunu7 2 роки тому

      @@WrathofMath yesterday i complete my real analysis xam..its too easy.. bcz i watch ur classes

  • @tiyabanda7555
    @tiyabanda7555 2 роки тому

    Clear explanation

  • @Puzzlers100
    @Puzzlers100 Рік тому

    Can a sequence be undefined at a point but still converge? 1/(n-1) would make a sequence where the first term is undefined (divison by zero) and the remaining terms would eventually converge to 0. Is this not a valid sequence?

    • @WrathofMath
      @WrathofMath  Рік тому

      Good question, I would just consider this sloppily written mathematics. A sequence is a function from the naturals to the reals, meaning there is a term assigned to position 1, 2, 3, etc. In the case you describe, the "undefined term" simply isn't a term in the sequence, and so really the sequence should be 1/n, since this describes the intended sequence without the weird ambiguity of a first "undefined" term. A sequence is a function, so it can't have any undefined values, since a function has one defined output for each input.

  • @Dupamine
    @Dupamine Рік тому

    I have the same question as @ezanabo < When u were explaining for n = 1,2,3,...,n u said that it's possible for those n to be bigger than 1+a please I don't understand how 🙏

    • @WrathofMath
      @WrathofMath  Рік тому

      Can you give me a timestamp?

    • @Dupamine
      @Dupamine Рік тому

      @@WrathofMath It is at 1:30. a-1

    • @WrathofMath
      @WrathofMath  Рік тому

      {1,2,3,4,5} is not a sequence because a sequence is a function from the naturals to the reals, which is to say it has infinitely many terms. If a sequence converges, as we've assumed ours converges to a, then for any epsilon>0, there is some point in the sequence a_N, after which all terms of the sequences are within epsilon of the limit a. So if epsilon = 1 for example, there is some a_N after which |a_n - a| < 1 for all n>N, which means -1 < a_n - a < 1 for all n>N.
      Since the set you mention is not a sequence, and so it certainly doesn't converge, this line of reasoning doesn't apply to it. Does that help?

    • @WrathofMath
      @WrathofMath  Рік тому

      On the other hand, if we consider the sequence 1, 2, 3, 4, 5, 5, 5, 5, ...
      Then it is true that 5 - 1 < a_n < 1 + 5 for all n>4. After this point, the sequence is within 1 of its limit, and in particular for this sequence it happens to equal its limit after this point. Remember the definition of convergence guarantees this after some point in the sequence, it certainly need not be true for every term of the sequence.

  • @munugotianusha5305
    @munugotianusha5305 2 роки тому

    Nice teaching 👌👌

    • @WrathofMath
      @WrathofMath  2 роки тому

      Thank you! Let me know if you have any questions, and check out my analysis playlist for more! ua-cam.com/play/PLztBpqftvzxWo4HxUYV58ENhxHV32Wxli.html

  • @hussainfawzer
    @hussainfawzer Рік тому

    What’s the software you’re using here ?

    • @WrathofMath
      @WrathofMath  Рік тому +1

      Notability for iPad! It's a wonderful app.

  • @PAWAN-SINGH-IIT-DELHI
    @PAWAN-SINGH-IIT-DELHI Рік тому

    Awesome

  • @avtaras
    @avtaras 11 місяців тому

    Might be a dumb question, but just because one can find a value ε = 1 such that ∀n>N, |xₙ − a| < ε, it doesn’t mean that the sequence converges? If one took epsilon to be arbitrary then it would make sense. So how can one just say epsilon is equal to 1 and proceed with the proof.

    • @WrathofMath
      @WrathofMath  11 місяців тому +3

      Thanks for watching and the question! It's because the sequence's convergence is not in question. We are assuming the sequence is convergent, and thus the for every n>N stuff must be true for ANY epsilon, so we are certainly at liberty to take a specific epsilon which is convenient for our proof, that the sequence is bounded.

    • @blackhole1222
      @blackhole1222 5 місяців тому

      thanks!
      @@WrathofMath